[Math] Question about the Continuum Hypothesis

logicset-theory

The Continuum Hypothesis hypothesises

There is no set whose cardinality is strictly between that of the integers and the real numbers.

Clearly this is either true or false – there either exists such a set, or there does not exist such a set.

Paul Cohen proved that the Continuum Hypothesis cannot be proven or disproven using the axioms of ZFC.

If we find a set whose cardinality lies strictly between that of $\mathbb{N}$ and $\mathbb{R}$, then we are done, we have disproven it. But it has been proven that we cannot disprove it, thus by contrapositive, we cannot find such a set. If we cannot find such a set, then we can only conclude that such a set does not exist (if it did exist, there must be a non-zero probability that we would find it, so given enough time we would – contradiction. □)

So I have proven that the Continuum Hypothesis is true – there does not exist such a set. But this is a contradiction because it has been proven that we cannot prove it either. So where did I go wrong?

Thanks!

Best Answer

Cohen's result is that from a certain set of axioms ($\mathsf{ZFC}$) we cannot prove the continuum hypothesis. Gödel's result is that from the same set of axioms, we cannot refute the continuum hypothesis. This only means that the set of axioms under consideration is not strong enough to settle this question. If you manage to exhibit a set of intermediate size, your argument necessarily uses axioms beyond those in $\mathsf{ZFC}$, or is not formalizable in first-order logic. Similarly if you manage to show that there is no such set. It is worth pointing out that there are standard axioms beyond those in $\mathsf{ZFC}$ that settle the continuum problem. These axioms are not universally accepted yet, but this illustrates that, as explained, their results are not absolute, but relative to a very specific background theory.

Related Question